Multivariable limits, how to show existence or non-existence

In summary, the student is trying to find a function that approaches 0 as x and y approach 1, but is having difficulty because the limit does not exist along one of the possible paths.
  • #1
mike1967
16
0

Homework Statement


lim(x,y)->(1,0) of ln((1+y^2)/(x^2+xy))


Homework Equations




The Attempt at a Solution



Used two paths,
x=1
y=0
both gave my lim=0
so I tried x=rsin y=rcos, in attempt to use ε-δ to prove it.

got to ln((1+r^2sin^2)/(r^2cos(cos+sin)))

not sure where to go from here.
 
Physics news on Phys.org
  • #2
Try going linear, with y = m(x - 1) instead.
 
  • #3
mike1967 said:

Homework Statement


lim(x,y)->(1,0) of ln((1+y^2)/(x^2+xy))

Homework Equations



The Attempt at a Solution



Used two paths,
x=1
y=0
both gave my lim=0
so I tried x=rsin y=rcos, in attempt to use ε-δ to prove it.

got to ln((1+r^2sin^2)/(r^2cos(cos+sin)))

not sure where to go from here.
I don't see that there's any difficulty as long as x→1 and y→0. ln(1/1) = 0
 
  • #4
My issue is in lecture my professor made it clear that finding any finite number of ways a function approached the same number did not prove that the lim was equal to that, in this case 0, because there are infinite number of ways (x,y) can approach the point. Does this make sense or did I misunderstand? Basically the only way he taught us to prove a lim existed was to use the ε-δ.
 
  • #5
mike1967 said:
My issue is in lecture my professor made it clear that finding any finite number of ways a function approached the same number did not prove that the lim was equal to that, in this case 0, because there are infinite number of ways (x,y) can approach the point. Does this make sense or did I misunderstand? Basically the only way he taught us to prove a lim existed was to use the ε-δ.
Yes, what your prof. said makes sense. I'm pretty sure that the functions that cause trouble are of indeterminate form, usually 0/0 . Very often the limit is being taken as (x,y)→(0,0) in which case using polar coordinates is often a big help.

For the problem in this thread, you have neither 0/0, and (x,y)→(1,0) rather than (0,0).

BTW: If the limit does not exist, then if you may be able to show that the limit is different along different paths.
 
  • #6
Well along the path x=y the limit blows up, 1/0, so then the limit does not exist?
 
  • #7
mike1967 said:
Well along the path x=y the limit blows up, 1/0, so then the limit does not exist?
The line y=x doesn't go through the point (1,0) .
 
  • #8
opps, ok. I think I solved it now.
The limit exists and is equal to 0
epsilon=r
delta=r
epsilon=delta.
 

1. What is a multivariable limit?

A multivariable limit is a mathematical concept that describes the behavior of a function as the input values approach a specific point in a multi-dimensional space. It is used to determine if a function has a finite value or tends towards infinity as the input values approach a given point.

2. How do you calculate a multivariable limit?

To calculate a multivariable limit, you must first determine the limit along each coordinate axis separately. This can be done by plugging in the coordinates of the given point into the function. If the limits along each axis are equal, then the multivariable limit exists. If they are not equal, the limit does not exist.

3. What is the difference between a one-sided and two-sided multivariable limit?

A one-sided multivariable limit only considers the behavior of a function as the input values approach a given point from one side. This is useful when dealing with functions that have discontinuities. A two-sided limit considers the behavior from both sides and is used to determine the overall limit at a point.

4. How do you show the existence of a multivariable limit?

To show the existence of a multivariable limit, you must prove that the limit along each coordinate axis is equal. This can be done by using the definition of a limit and plugging in the coordinates of the given point into the function. If the resulting limits are equal, then the multivariable limit exists.

5. How can you determine the non-existence of a multivariable limit?

The non-existence of a multivariable limit can be determined by showing that the limit along at least one coordinate axis is not equal to the others. This can be done by using the definition of a limit and plugging in the coordinates of the given point into the function. If the resulting limits are not equal, then the multivariable limit does not exist.

Similar threads

  • Calculus and Beyond Homework Help
Replies
9
Views
1K
  • Calculus and Beyond Homework Help
Replies
13
Views
2K
  • Calculus and Beyond Homework Help
Replies
10
Views
820
  • Calculus and Beyond Homework Help
Replies
2
Views
1K
  • Calculus and Beyond Homework Help
Replies
7
Views
971
  • Calculus and Beyond Homework Help
Replies
5
Views
799
  • Calculus and Beyond Homework Help
Replies
5
Views
858
  • Calculus and Beyond Homework Help
Replies
6
Views
543
  • Calculus and Beyond Homework Help
Replies
2
Views
170
  • Calculus and Beyond Homework Help
Replies
6
Views
2K
Back
Top